Vous êtes sur la page 1sur 17

2.

ABC is an obtuse angled triangle whose sides


are 7 cm, 16 cm and y cm where y is an integer.
Find the number of possible values of y.
(1) 9 (2) 10 (3) 6 (4) 16 (5) 17

3.

15. How many 5 - digit numbers, comprising only the


digits 4 and 5 are divisible by 3?
(A) 10
(B) 11
(C) 12
(D) More than 12

PQRS is a square. M and N are the midpoints of


PS and QR respectively. X and Y are points lying
on the line joining M and N inside PQRS, such
that PXS = QYR = 60. Find the ratio of the
area of the hexagon PQXRSY and the remaining
area inside PQRS.

16.

(1) 4

(2) 2 3 + 3

(4) 1 +

(5)

14. All possible words are formed using all the letters
in the word EQUATION exactly once and are
arranged such that no two vowels and no two
consonants are in alphabetical order. How may
words are there?
(A) 56
(B) 140
(C) 720
(D) 4914

4.

There is a rectangular grid consisting of unit cells


as shown in the figure given below. One needs to
travel from point A to point C. One can travel only
to the right and upwards. In how many ways can
one go from point A to point C without traveling
via B?
(A) 3915
(B) 3150
(C) 2520
(D) 6434

19. Three, four and five points are selected on the


sides AB, BC and CD respectively of ABC.
None of the 12 points coincide with A, B and C.
How many triangles can be formed by using
these 12 points as vertices?
(A) 220
(B) 217
(C) 210
(D) 205

1.

PQR is a triangle. PQ = 19.5 cm and QR = 11 cm.


PS is the altitude of PQR of length 5 cm. C is a
circle circumscribing the triangle PQR. Find the
radius (in cm) of C.
(1) 18.05
(2) 28.85
(3) 27.25
(4) 21.45
(5) 31.25

2
3

3
2

5.

The radius of the base of a right circular cone is


6 cm and the height is 21 cm. The cylinder having
the maximum possible total surface area is
placed inside the cone such that one of its flat
surfaces rests on the base of the cone. Find the
total surface area of the cylinder, in sq.cm.
1321
1383
1353
(3)
(1)
(2)
8
8
8
1473
441
(5)
(4)
8
5

6.

C1 and C2 are two equal circles whose centers


are M and N respectively. They intersect at X and
Y. Neither of the centers lies inside the other
circle. If XMN = , what is the range of possible
value of ?
(1) 0 30
(2) 0 45
(3) 0 60
(4) 0 75
(5) 0 90

7.

Two identical circles intersect each other.


The radii of the circles and the distance joining
the centres of both the circles are in the ratio of
1 : 3 . Find the ratio of the area of the region
common to both the circles, to that of a circle.

Geometry & Mensuration


Directions for questions 1 to 40: Select the correct
alternative from the given choices.

3 +1

Each of two circles passes through the centre of


the other. The radius of either circle is 2 cm.
Find the area of the intersecting region (in sq.cm).
8
4
+2 3
3
(2)
(1)
3
3
8
8
(4)
+ 3
(3)
2 3
3
3

(5)

17. How many three-digit odd number can be formed


such that if 2 is one of the digits, the following
digit is 4?
(A) 5
(B) 365
(C) 300
(D) 372
18. A coaching institute was to send 9 parcels for its
postal students, four of whom were from Bihar
and the remaining five were from Delhi. The
dispatch clerk made a mistake in addressing the
parcels. He addressed them in such a way that
no student received the correct parcel although
all the parcels for Bihar were sent to Bihar and all
the parcels for Delhi were sent to Delhi. Find the
total number of ways in which he could have
addressed those 9 parcels.
(A) 396
(B) 2737 (C) 475
(D) 89496

(3) 2 3 1

8.

(A)

3
4

(B)

3 2 3
12

(C)

3
6

(D)

2 3 3
6

A large number (greater than 1200) of equilateral


triangles of side 2 units are available to form
equilateral triangles. Equilateral triangles of sides
16 units, 18 units, 30 units are formed using
the available triangles. Find the total number of
triangles used.
(A) 1050 (B) 1100 (C) 1150 (D) 1200

9.

Find the side of the smallest equilateral triangle


that can contain a semicircle of radius 8 3 cm.
(in cm)
(A) 16 3

(B) 32

(C) 32 3

(D) 40

10. In the figure below, ABC is


equilateral. Three identical
circles are shown in it. Each
circle touches the other two
circles and has two of the
sides of the triangle being
tangents to it. Find the ratio of the side of the
triangle and the radius of each circle.

( )
2(1 + 3 )

(
)
2(1 + 3 3 )

(A) 2 2 + 3

(B) 2 1 + 2 3

(C)

(D)

11. N points are marked on the circumference of a


circle. The number of triangles which can be
drawn by using these points as vertices is 210
more than the number of triangles which can be
drawn by using all but one of these points as
vertices. Find N.
(A) 21
(B) 22
(C) 23
(D) 24
12. Let X denote the product of the sides of a triangle.
Let Y denote the product of its semi-perimeter,
Y
circumradius and its inradius.
=
X
1
1
(B)
(A)
6
4
1
(C)
(D) Cannot be determined
3

13. In the figure above, PQRS P


is a square. A sector of a
circle with centre S is
shown.
TQVU
is
a
rectangle such that TU = 9
and UV = 32. Find PS.
(A) 60
S
(B) 65
(C) 70
(D) Cannot be determined

T
U

Q
V

14. Two intersecting circles are said to be orthogonal,


if a pair of tangents drawn to them at any point of
intersection are perpendicular to each other.
C1 and C2 are orthogonal circles. Each of them
has a radius of 8 cm. Find the area of the region
common to both the circles. (in sq cm)
(A) 16(2 1)
(B) 16(2 3)
(C) 32( 1)
(D) 32( 2)
15. On the periphery of a square grass field ABCD,
two poles are fixed. One of them is fixed at the
midpoint of side AB while the other one is fixed at
the midpoint of side BC. Side of ABCD is 16 m.
A cow is tied to one of the poles and another is
tied to the other pole. If each cow is tied with a
8 m long rope, then find the ungrazable area in
the field. (in sq. m)
(A) 256 32
(B) 216 32
(C) 192 32
(D) 240 32

16. PQR is an isosceles triangle with PQ = QR. QA is


the median to PR. B is a point on QR (or QR
extended) such that AQ = AB. PQA = 30.
Find ABQ.
(A) 20
(B) 30
(C) 40
(D) Cannot be determined
17. A triangle has an area of 960 sq m. Two of its
sides are 68 m and 32 m. Find its third side if it is
given that the triangle is right angled (in m).
(A) 52
(B) 58
(C) 62
(D) None of these
18. A right circular cone has a radius of 18 cm and a
height of 54 cm. From it, a right circular cylinder
of radius 15 cm was cut. Find the volume of the
conical part of the remaining solid (in cubic cm).
(A) 4540 (B) 4375 (C) 4780 (D) 3375
19. Two plastic cubes each of which have an integral
side (in cm), have the sum of their lateral surface
areas equal to 468 sq cm. The sum of their
volumes is 945 cubic cm. Find the volume of the
hemisphere whose radius equals the sum of the
edges of the two cubes (in cubic cm)
(A) 2150 (B) 2250 (C) 2325 (D) 2400
20. In ABC, AB = 32, BC = 24 and AC = 40. Using
B as a centre, a circle is drawn. This circle cuts
AB and BC at D and E respectively. The radius of
this circle equals the circumradius of ABC.
Find AD : EC.
(A) 3 : 1
(B) 2 : 1
(C) 3 : 2
(D) 5 : 2
21. There are 13 spherical balls, each of radius
3 2 . Nine of the balls are arranged in layer in a
3 x 3 square formation. Four balls are placed in a
second layer over the four depressions which are
formed. Find the height of the smallest cuboid
which can enclose these balls.

(A) 6 + 6 2

(B) 6+6 3

(C) 2 3 +2 6

(D) 3 2 +6

22. Four points are chosen at random in a region


comprising an equilateral triangle of side
4 3 and its interior. Which of the following is
true?
(A) There are at least two points P, Q such that
PQ 4.
(B) There are at least two points P, Q, such that
PQ 4.
(C) There are at least two points PQ, such that
PQ > 4.
(D) There are at least two points PQ, such that
PQ < 4.
23. In the figure below, PQRS is a rectangle with
PQ = 9 and PS = 6. MNOP is a smaller rectangle
with MN parallel to PQ. Also the lengths of MN
and MP are integers and A, B, C, D are the
midpoints of the sides of the rectangle MNOP
shown in the figure. The area of the shaded
region is 162/3% of the area of the rectangle
PQRS. Find the length of AB.

(A)

N
B

D
S

29. In the figure below, circles with centres A, B and


C have equal radii and similarly circles with
centres D, E and F have equal radii. Find the ratio
of the radii of circles with centres at D and G.

(B) (3 / 2) 5

25. In the triangular field PQR, PQ = 54 m, QR = 240 m


and Q =900. [PQ, AB, CD, EF, GH, and IJ are
all equispaced and all the lines are perpendicular
to QR. Also HI = IR]. If one moves along
PQABCDEFGHIJR, then what is the total
distance covered (in meters)?
P

B
C

F
G

Q A D E

(A) 270m
(C) 351m

H I

(B) 594m
(D) None of these

26. The maximum number of acute angles in a


convex octagon is
(A) 1
(B) 2
(C) 3
(D) None of these
27. There are 6 boys B1, B2, B3, .B6 standing
at the vertices A to F respectively of a regular
hexagon. They start walking simultaneously
along the perimeter in the clockwise direction with
speeds in the ratio
1 : 2 : 3 : 4 : 5 : 6. When
B6 completes 3 full rounds, which are the vertices
at which there are no boys?
(A) A, C
(B) B, E
(C) A, C, E
(D) B, D
28. On square ABCD, points P, Q, R and S are on
sides DA, AB, BC, CD such that AP = AQ = CR =
CS X and Y are points on AB and CD
respectively, which are equidistant from P and R.
XB + YD
Find
AP + CR
(A) 0.5
(B) 0.75
(C) 1
(D) Cannot be determined

(A)

13 + 10 3
11

(B)

17 + 10 3
11

(C)

19 10 3
11

(D)

10( 2 + 2)
11

30. Which of the following statements is/are true?


() A triangle exists with altitudes measuring 6,
9 and 15
() A triangle exists with altitudes measuring 4,
9 and 12 which of the following is true.
(A) Only
(B) Only II
(C) Both and
(D) Neither I or
31. A, B, C and D are four friends who are standing
at the four corners of a rectangular field in the
anticlockwise order. They decide to meet at a
point P inside the field. If the shortest distance of
the point P from the initial position of A, B, C, and
D are 20 m, 40 m, 60 m and x m respectively,
then find the value of x.
(A) 30 2

(C) 2 5
(D) Cannot be uniquely determined
24. There are two concentric circles with their centre
at O. Chord AB of the outer circle is tangent to
the inner circle. If the measure of the line AB is
40 cm and the radii of both the inner and the
outer circle are integral number of centimeters,
then which of the following cannot be the length
of the diameter of the outer circle?
(A) 50
(B) 58
(C) 96
(D) 104

(B) 20 6

(C) 40 3

(D) 50

32. When a rectangle ABCD (with AB < BC) is folded


such that the vertex C touches vertex A, the
length of the crease XY formed is equal to the
length of the rectangle. How many times the
length of the rectangle is its breadth?
2 + 1

(A)
2

5 + 1

(C)
2

1
2

2 1

(B)
2

1
2

5 1

(D)
2

1
2

1
2

33. Two perpendicular chords PQ and RS intersect at


T PT= 4 and QT=18. If TS is 2 times TR, then
find the radius of the circle
(A)

110

(B) 2 30

(C)

130

(D) 5 6

34. On side AB of rectangle ABCD, P is a point such


that APD : DPC: CPB = 2 : 5 : 5. Find AB :
BC.
(A) 3
(C) 2

(B) 2 3 2
(D) Cannot be determined

35. A cuboid has a square base. The length of its


longest diagonal is

2502 . All dimensions of the

cuboid are integral. How many such cuboids are


possible?
(A) 1
(B) 2
(C) 3
(D) 4
36. In a cyclic quadrilateral, the diagonals AC and BD
intersect at P at right angles. If PA = 8, PC = 9
and the radius of the circumcircle of ABCD is
81.25 , then find PD given that where PD < PB.
(A) 12
(B) 9
(C) 8
(D) 6

37. In trapezium ABCD, the diagonals AC and BD


intersect at P. Three times the area of the
pentagon APBCD is equal to 7 times the area of
PCD. Find AP : PC.
3
4
3
2
(C)
(D)
(A)
(B)
2
4
3
3

38. The semiperimeter of a right angled triangle is


60 and the inradius is 8. Find the length of the
smaller leg.
(A) 30
(B) 20
(C) 24
(D) 18
39. There are two vertical poles AB and CD where
B and D are on the ground. AB = 4, CD = 7.5,
BD = 11.5. On line BD, P is a point where the
points A and C subtend a right angle and Q is a
point that is equidistant from A and C. Find PQ.
(A) 3
(B) 3.5
(C) 4
(D) Cannot be determined

3. Nagu bought a new bike and went for a drive,


liding at a uniform speed on a highway. At 9 am
he passed a milestone. He turned his head back
and read the number on the milestone.
He continued driving and at 10 am, he passed
another milestone. Again he turned his head back
and saw that the number was the reverse of the
number on the first milestone. He continued
driving and at 11 am he passed a third milestone.
Again, he turned his head back to note the
number. The sum of the digits of the number on
this milestone was equal to the sum of the digits
of the number on the first milestone. Which of the
following is the speed at which he travelled?
(Assume that the numbers he saw were
increasing).
(A) 9 km/hr
(B) 45 km/hr
(C) 18 km/hr
(D) Cannot be determined.

Equations, Ratio, Proportion, Variation


Directions for questions 1 to 6: Select the correct
alternative from the given choices.
1.

Amar, Bhuvan, Chetan and Dinesh are


four friends. Amar has m marbles with him.
He gives Bhuvan 1 less than half the number of
marbles he has. Then he gives Chetan 1 less
than half the remaining number of marbles he
has. Finally, Amar gives Dinesh 1 less than half
the remaining number of marbles he has and is
left with 4 marbles. Which of the following best
describes the value of m?
(1) 1 m 4
(2) 5 m 9
(3) 9 m 13
(4) 10 m 14
(5) m 14

2.

If a

3.

If 3a + 5b + 7c = 1.25 k and 2a + b + 3c = 0.75 k,


then 7b + 5c is what percentage of k?
(A) 25%
(B) 50%
(C) 35%
(D) 75%

4.

There are some two rupee coins and five rupee


coins in a bag. If the number of five rupee coins is
tripled, then the amount in the bag is increased
by 75%. Which of the following can be the
number of five rupees coins in the bag?
(A) 13
(B) 20
(C) 18
(D) 32

5.

There are 100 questions in a test paper. Four marks


are awarded for each right answer and two marks
are deducted for each wrong answer. If Abhilash
attempts more than 85 questions and get
70 marks, What is the minimum number of
questions that he could have answered correctly?
(A) 40
(B) 38
(C) 39
(D) 41

40. In triangle PQR, Q =900 and PQ = QR. D is a


point on the same side of PR as Q such that PR
=
(A)
(B)
(C)
(D)

2 QD. Find the measure of angle PDR.


221/2
30
45
Cannot be uniquely determined

Special Equations
Directions for questions 1 to 3: Select the correct
alternative from the given choices.
1.

2.

In a certain country, the currency used was called


femto. Raju had his lunch in one of the hotels in it.
His bill came to 47 femtos. He had notes in only
3 denominations 1-femto, 2-femto and 20-femto.
In how many ways can he settle the bill?
(1) 24 (2) 18 (3) 48 (4) 16 (5) 42
A bank teller was making the payment for a
cheque presented by Mohan. As he was in a
confused state of mind, he transposed the rupees
and paise and hence gave more than what he
should have. Mohan left the bank and bought a
biscuit from a nearby store for `1.50.
The amount remaining with him was 4 times the
amount on the cheque. The amount remaining
with him must have been between
(1) `70 and `71
(2) `79 and `80
(3) `85 and `94
(4) ``93 and `94
(5) `98 and `99

1
and b c2, then find the value of
b
9a + 5c, when b = 27. Given that when b = 3 then
a = 4 and c = 6.
(A) 94
(B) 49
(C) 76
(D) 38

6.

Arun, Varun and Kiran have a total of `8,000 with


them. They spend `50, `100 and `200
respectively, Now they have a money is the ratio
of 14 : 22 : 15. What is the amount with the Varun
initially?
(A) 2150
(B) 3400
(C) 3300
(D) 2100

5.

In an x-metre race, A beats B by 180 m and


C beats by 351 m. In the same race, B beats
C by 198 m. Find x?
(A) 1200 (B) 1000 (C) 1122 (D) 1320

6.

Towns A and D are 36 km apart. Three friends,


Tarun, Varun and Arun start together from
A towards D. While Arun sets off on foot, Varun
takes Tarun along on his bike and travels at
45 km/hr. He drops Tarun at a point C and turns
back for Arun. He meets Arun at a point B, and
then turns back towards D. All the three friends
reached D together. If both Arun and Tarun
walked at 5 km/hr, how long do the 3 take to
cover AD?
1
2
13
(A) 3
(B) 3
(D) 1
(C) 3
9
15
3

Time and Distance


Directions for question 1: Select the correct
alternative from the given choices.
1.

P and Q are 2 stations. Raju plans to drive from


city R, located 840 km directly to the north of Q,
210
to Q, at a speed of
kmph so that he can
3
catch a train arriving there from P. The train
leaves P at 9 a.m. and travels at a speed of
84 3 kmph. P is between east and south-east
of R with RQ at 60 to RP. Also P is between the
north and the north-east of Q with PQ at 30 to
RQ. If Raju has to reach P at least 25 minutes
before the train, then find the approximate latest
time at which he should start from R.
(1) 6:50 a.m. (2) 6:40 a.m. (3) 6:20 a.m.
(4) 6:30 a.m. (5) 7:00 a.m.

Directions for questions 2 and 3: Answer these


questions based on the information below.
Cities P and Q are 4800 km apart. P is to the west of
Q. Both cities are in different time zones Best
Airlines was an airline which operates non-stop
flights between the cities. All its planes cruise at the
same speed in both directions. However, the effective
speed of any plane is influenced by a steady wind
blowing from west to east at 100 kmph. The table
below shows the departure time of the planes from
each city and their arrival time at the other city.
(Given below are local times of the respective cities)
Departure
City
Time
P
7:00 a.m.
Q
6:00 p.m.

City
Q
P

Arrival
Time
5:00 p.m.
4.00 a.m.

2.

Find the time in P when the plane landed in Q.


(1) 4:00 p.m.
(2) 3:00 p.m.
(3) 3:30 p.m.
(4) 2:30 p.m.
(5) Cannot be determined

3.

Find the cruising speed of the plane (in kmph).


(1) 600
(2) 550
(3) 500
(4) 450
(5) Cannot be determined

Directions for questions 4 to 8: Select the correct


alternative from the given choices.
4.

A starts from point P at 9:00 a.m. and travels east


at 45 km/hr. B starts a bit later from P and travels
south at 30 km/hr for 48 minutes. At that instant,
he is 74 km from A. When does B start from P?
(A) 9:42 a.m.
(B) 9:45:20 a.m.
(C) 9:48 a.m.
(D) 9:51:40 a.m.

7. Two friends P and Q start simultaneously from


the opposite ends of a race track AB of length
150 m at speeds 60 m/s and 40 m/s respectively.
P starts from A and Q starts from B. Once each
reaches an end, he immediately turns back and
moves towards the other end. They keep moving
to and from between the two ends. Find the
difference between the distance covered by P
and Q by the 8th meeting.
(A) 300 m (B) 450 m (C) 600 m (D) 1200 m
8. Ajay and Sanjay start simultaneously from the
same point on a circular track. If they travel in
opposite directions, they meet at 7 distinct points
on the track whereas if they travel in the same
direction, then they meet at n distinct points on it
where n is a prime number. If Ajay is faster than
Sanjay and Sanjay's speed is P% less than that
of Ajay, then which of the following can be a
value of P?
(A) 25
(B) 50
(C) 75
(D) 831/3

Quadratic Equations
Directions for questions 1 and 2: Select the correct
alternative from the given choices.
1.

Three consecutive positive integers are taken in


descending order. The first, second and the third
are raised to the first, second and third powers
respectively. The powers are added and the
result is multiplied by 3. The square root of the
result is found to be the sum of the original
integers. The least of the integers is denoted by
L. Which of the following holds true?
(1) 1 L 4
(2) 5 L 8
(3) 9 L 12
(4) 13 L 15
(5) L > 15

2.

The roots of the equation x3 px2 + qx r = 0 are


a, b and c, which are consecutive integers.
Find the least possible value of q.
(1) 0

(2) 1

(4) 1

(5) 2

(3)

Directions for questions 3 and 4: Answer these


questions using the information below.
Let g(x) = px2 + qx + r, where p, q and r are constants
and r 0. One root of g(x) = 0 is 4 and 8g(3) = 3g(6).
3.

4.

Find the other root of g(x) = 0.


(1) 1
(2) 3
(3) 6
(4) 2
(5) Cannot be determined
Find the sum of p, q and r.
(1) 21
(2) 20
(3) 19
(4) 18
(5) Cannot be determined

Directions for questions 5 and 6: Answer these


questions based on the given information.

12. A certain number of cups of tea are available for


`90. If the price of each cup increases by `1.50,
the number that can be bought for the same
amount decreases by 10. Find the actual cost of
each cup of tea (in rupees).
(A) 3
(B) 3.50
(C) 4
(D) 4.50
13. The expression x4 + y4 2x2 y2 32x2 32y2 +
256 is completely factorized into real factors.
Which of the following statements about the
factors is true?
(A) There are 2 irreducible quadratic factors.
(B) There is a cubic factor and a linear factor.
(C) There are 4 linear factors.
(D) None of these

Raju makes and sells an item in a market every day.


He sells each unit of that item at `106. The cost of
producing x units per day is 200 + px + qx2, where p
and q are constant. If Raju increased his daily
production from 10 units to 15 units, his daily
production cost would increase by 888/9%. If he
increased his daily production from 15 units to
20 units, his daily production cost would increase by
6412/17%. Assume that there is a high demand for the
item and that Raju can sell whatever he produces.
He wishes to maximize his profit.

14. The expression ax2 + bx + c takes a maximum


value of 5 at x = 1 and takes the value of -1 at
x = 0. Find the value of the expression at x = 5.
(A) 89
(B) 91
(C) 210
(D) 211

5.

Rajus daily production must be (in units).


(1) 10 (2) 15 (3) 8 (4) 5 (5) 12

6.

Find the maximum daily profit that Raju can


obtain (in Rs).
(1) 400
(2) 425
(3) 350
(4) 375
(5) 184

16. If the sum of the roots (not necessarily real) of a


quadratic equation is 6 and the sum of the
squares of the roots is 16, then find the product of
the roots.
(A) 8
(B) 10
(C) 9
(D) Such an equation doesn't exist

Directions for questions 7 to 18: Select the correct


alternative from the given choices.
7.

E(y) is a quadratic expression. It has the


minimum value of 1 when y = 3 and E(2) = 2.
Find E(4).
(1) 40 (2) 45 (3) 50 (4) 55 (5) 60

8.

If the equations ax2 + bx + c = 0; bx2+ cx + a = 0


have one root in common, then which of the
following is definitely true?
(A) a3 + b3 = 3abc
(B) a3 + b3 + c3 = 3abc
3
3
3
(C) a b c = 3abc
2
2
2
(D) a + b + c = 2ab + 2bc + 2ca

9.

If , and are the roots of the equation


3
2
x + 2x 5x 6 = 0 and , , are the roots of
the equation x3 + px2 + qx + r = 0; then the value of
p is
(A) 6
(B) 5
(C) 5
(D) 6

10. Let and be the roots of a quadratic equation


and = 9 and || || = 5 then find the
product of roots.
(A) 14
(B) 21
(C) 16
(D) 7
11. What is the minimum value of the square of the
difference of the roots of the quadratic equation
x2 (k + 7) x (3k 15) = 0?
(A) 160
(B) 180 (C) 160 (D) 180

15. The roots of the equation x3 + ax2 + bx + c = 0,


where c > 0, are k, kr, kr2 where k and r are
integers. If the sum of the squares of the roots is
364, then find the value of c
(A) 180
(B) 72
(C) 108
(D) 216

17. The roots of x3 21x2 + px 280 = 0 are in


arithmetic progression. Find the value of p.
(A) 138
(B) 128
(C) 118
(D) Cannot be determined
18. If cubic equation ax3 + bx2 + cx + d = 0 has two
positive roots where a, b, c, d are real and d0,
then which of the following is true?
(A) c and b are of opposite signs.
(B) a and d are of the same signs.
(C) b and c are of the same sign.
(D) a and d are of opposite signs.

Progressions / Series
Directions for questions 1 and 2: Select the correct
alternative from the given choices.
1.

Find the number of common terms of the


sequences 24, 29, 34, .. 474 and 25, 29, 33,
, 485
(1) 20 (2) 21 (3) 25 (4) 24 (5) 23

2.

Nilgiris coffee cost `(110 + 0.2N) per kg on the


Nth day of 2006 where 1 N 200. Its price

th

remained constant that year from the 200 day.


Coorg coffee cost `(97 + 0.25N) per kg on the
Nth day of 2006 where 1 N 365. Find the date
in 2006 on which the prices of the two varieties of
coffee were equal.
(1) June 25
(2) August 10
(3) July 31
(4) August 20
(5) July 21

Directions for questions 3 and 4: Answer these


questions based on the given information
M and N are positive quantities. Let g1 = M and h1 = N.
When x is even, gx = Mhx 1 and hx = Mgx 1
When x is odd, gx = Nhx 1 and hx = Ngx 1
3.

9.

9
10. If S =
7
3
(A)
2

x 1

(3) M(MN)

(M + N)

1
x 1
2

12. If a, b, c, are 3 positive numbers in geometric


progression and a12 = b15 = cx, then x =
(A) 18
(B) 20
(C) 21
(D) 24

(M + N)

(4) MM 2 (M + N)
1

(5) M(MN) 2

4.

x 2

13.

(M + N)

1
3
and N = , find the least odd x for which
4
4
gx + hx < 0.02?
(1) 3 (2) 5 (3) 7 (4) 9 (5) 11

If M =

Directions for questions 5 to 17: Select the correct


alternative from the given choices.
5.

If

1
+
10.11.12.13

2149
5616600
1429
(C)
5666100

6.

1
, then the value
99.100.101.102

(B)

Find the sum of all the five


leave a remainder of either
by 8.
(A) 659100000
(B)
(C) 1318200000
(D)

digit numbers which


4 or 6 when divided
561900000
1237522500

7.

If the sum of the first n terms of an arithmetic


progression in 2400 and the sum of next n terms
as 7200, then find the ratio of first term and
common difference.
(A) 3 : 2
(B) 2 : 1
(C) 1 : 2
(D) 2 : 3

8.

If the nth term of a series is given by tn =


t
then 31 is
t 27
(A) 0
(C) 1

t n 1 + t n + 1
2

15. The nth term tn of a sequence is defined by tn = tn-1


tn-2. If t1 = 3, t2 = 4, and tn tm is a multiple of 10,
which of the following is not a possible value of
(m, n)?
(A) (10, 16)
(B) (16, 10)
(C) (7, 1)
(D) (8, 2)
16. The sum of n terms of an arithmetic progression,
starting with the 11th term is directly proportional
2
to n . The sum of 2 of the terms of the arithmetic
progression is 0. Which of the following cant be
either of these two terms?
th
th
st
th
(A) 5
(B) 11
(C) 21
(D) 16
17.

1(32 ) + 2( 42 ) + 3(52 ) + .... + 10(122 )


=
3(12 ) + 4(22 ) + 5(32 ) + ... + 12(102 )

(A)
(B) 1
(D) None of these

1
1
1
1
1
1
1
+. . . +
+

+
1
6
99
4
5
3
2

1
, the value of S is also equal to
100
1
1
1
1
1
+
+... +
+
+
(A)
1
25
3
4
2
1
1
1
1
+ ... +
(B)
+
+
53
100
51
52
1
1
1
1
+ ...
+
+
(C)
52
100
50
51
(D) None of these

1
+
11. 12. 13. 14

1249
5666100
4219
(D)
5616600

1
1
1
1
is equal
+
+ . . . +
+
13.14.15
1 .2 .3 2 .3 .4 3 .4 .5
to
59
1
26
31
(D)
(C)
(A)
(B)
105
7
105
210

14. If S=

of k is
(A)

126

ad infinitum, S =
124
5
(D) 3
(C)
3

1 3
5
7
9
27
and
+ +
+
+
+ ... +
2 6 12 20 30
210
1 1 1
1
S2 =
, then find the value of
+ + + ... +
2 3 4
14
2S2 S1
4
3
1
3
(D)
(B)
(C)
(A)
2
2
5
4

(1) M(MN) 2 (M + N)
(2) MN 2

28 65

26 63
1
(B)
2

11. If S1 =

Which of the following equals gx + hx when x is even?


1

Find the sum of the first 40 terms of the series


2 + 3 + 4 + 6 + 8 + 9 + 16 + 12 + . . .
21
21
(A) 6
(B) 2 + 628
20
20
(C) 2 + 628
(D) 6 + 629

4784
3795

(B)

15
11

(C)

29
23

(D)

6
5

Functions
Directions for questions 1 to 11: Select the correct
alternative from the given choices.
1.

G(x) is a function satisfying

G(a)
a
= G for all
G(b)
b

1
, find G(9).
9
1
(2)
(3) 81
9
(5) Cannot be determined

real a, b If G(3) =
1
81
(4) 9

(1)

2.

G(y) is a function satisfying the following conditions.


x

G(1) = 5400 and G(y) = X G(X). where X is a


y =1

natural number greater than 1. Find G(6).


2400
1800
2100
(3)
(1)
(2)
7
7
7
1500
(5) 240
(4)
7

3.

If f(xy) = f(x) + f(y) + f(x) f(y)

then the value of f(1/6) is


17
2
9
(B)
(C)
(A)
3
8
9

4.

5.

9. The continuous function f(x) satisfies


following conditions x
f(x + 2) f(x) = 4 for x 2
f(x + 2) + f(x) = 6 for 0 < x < 2
and f(x + 2) f(x) = 4 for x 0
also f(0) = 5
Which of the following is f(x)?
(A) 2x 4 + 1
(B) 2x + 5

(C) 4 x 2 + 3

17
8

x
, then the value of f(f(f(f(f(f( 3))))) =
If f(x) =
1+ x
3
6
5
4
(D)
(B)
(C)
(A)
37
26
25
19

If f(x +2) = f(x) + 7 and f(1) = 2; f(2) = 5 then the


ratio of f(150) to f(75) is
(A) 523 : 261
(B) 253 : 621
(C) 427 : 673
(D) None of these

th

x 1 n
, f (x) = f1[fn-1 (x)], find f8 (x)
7x + 2
2x + 1
2x 1
(B)
(A)
7x 1
7x + 1
2x + 1
2x 1
(C)
(D)
7x 1
7x 1

11. If f1(x) =

Averages
Directions for questions 1 to 5: Select the correct
alternative from the given choices.
1.

Twelve years ago, the average of the ages of the


members of a joint family having ten members
was 25 years. Four years later a member aged
50 years died and a child was born in the family
that year. Four years after that, another member
aged 50 years died and another child was born.
Find the present average age of the members of
the family (in years).
(1) 26 (2) 27 (3) 28 (4) 29 (5) 30

2.

There are two groups (A & B) of children in a joint


family. There are 3 more children in B as
compared to A. While the average age of the
children in A is 6 years more than that of B, the
combined age of the children in A is less than
that of B. After 4 years, the difference in the
combined ages of group A and group B will
double. If there are less than 10 children in the
family, which of the following can be the average
age (in years) of the children in B?
(A) 9
(B) 8
(C) 7
(D) 5

3.

The number and the average weights of different


groups of children are given below:
Number
Average Weight (Kg.)
M
40
M+2
45
M+2
50
7
60
If the average weight of the entire class is 50 kg.
Find the total number of students.
(A) 20
(B) 21
(C) 22
(D) 23

6. If f (x + y) = f(x) + f(y) , where x 1, y 1 and f(7) =


24.5, then find the value of f(1) + f(2) + f(3) + f(4) +
f(5) + f(6) + f(7).
(A) 63
(B) 84
(C) 98
(D) None of these
7. If f(x+1) f(x) = ax + b, then which of the
following is true about the graph of y = f(x)?
(A) A line with a slope of a
(B) A line with a slope of b
(C) A parabola
(D) None of these
8.

Consider the function


f(x) = (x + 3)p (x + 2)q (x - 1)r (x 3)s, where p. Q.
R. s are nonnegative integers.
f(3.5) = a, f(2.5) = b, f(0.5) = c
f(1.5) = d, f(3.5) = e
b
Also, ab > 0,
<0, cd > 0 and d + e = d e .
c
Which of the following can be the value of
(p, q, r, s)?
(A) (2, 3, 5, 1)
(B) (4, 3, 2, 0)
(C) (3, 2, 5, 1)
(D) (0, 1, 2, 3)

(D) 2 x 4 + 1

10. If f(x + 1) f(x) = x and f(x) is an n degree


expression in x, then n =
(A) 2
(B) 3
(C) 4
(D) Cannot be determined

10
2
,
and f(6) =
3
9

(D)

the

4. There are two vessels P and Q, P containing 120


L of milk and Q containing 120 L of water. In the
first operation 30 L is removed from P and poured

into Q and then 30 L from Q is poured back into


P. Like this, one more operation takes places i.e.
transferring 30 L from P to Q and then
transferring 30 L from Q to P. What is the ratio of
milk and water in P, finally?
(A) 2 : 1
(B) 13 : 6
(C) 15 : 7
(D) 17 :8
5. Dinku Beora was a chronic alcoholic and
because of persistent, health problems he
decided to quit drinking. He devised an
ingenuous way of doing so. He bought a 750 ml
goodbye bottle of Old Monk. On the first day, he
drank 5% of the contents in it and replaced that
quantity with water. Next day he drank 10% of the
contents in the bottle and replaced it with water.
th
Like this he continued on the 19 day he drank
95% of the contents in the bottle and replaced it
with water and on the 20th and last day he drank
the entire contents in the bottle. Find the ratio of
the total quantity of alcohol and water that he
drank in the entire process.
(A) 2 : 21
(B) 2 : 19
(C) 3 : 28
(D) None of these

Quant Based Reasoning


Directions for questions 1 and 2: Answer these
questions based on the following information. P, Q, R,
S and T are 5 horses. They participated in a race.
The following are the rules of the race.
(1) A person who bets on the winning horse gets
4 times the bet amount.
(2) A person who bets on the horse coming second
gets 3 times of the bet amount.
(3) A person who bets on the horse coming in third
gets back his amount.
(4) Other persons lose their amount.
Mohan had placed his bets on Q, R and T.
The amounts he bet on Q, R and T were `2000,
`4000 and `6000 respectively. He ended up with no
gain and no loss.
1.

2.

Which of the following is not possible?


(1) At least two horses finished before P.
(2) There were three horses between Q and P.
(3) T finished last.
(4) S came in second.
(5) There were three horses between T and R.
Suppose S finished in the fourth position. Then
which of the following is not possible?
(1) P finished first.
(2) One horse finished between Q and R.
(3) Q came in second
(4) T finished last.
(5) R came second.

Data Sufficiency
Directions for questions 1 and 2: Answer these
questions based on the instruction below.
In the questions below, each question has two
statements A and B following it. Mark your answer as

(1) if the question can be answered from A alone but


not from B alone.
(2) If the question can be answered from B alone but
not from A alone.
(3) if the question can be answered from A alone as
well as from B alone.
(4) if the question can be answered from A and B
together but not from any of them alone.
(5) if the question cannot be answered even from A
and B together.
A certain number of players participated in a
tournament, played according to the following
rules. The number of players at any stage is
denoted as N.
N
pairs.
(i) if N is even, the players are grouped into
2
The players in every pair play against each other.
The resulting winners move on to the next round.
(ii) If N is odd one player is allowed to move on to
the next round. He is said to be given a bye. The
N1
remaining N 1 players are grouped in to
2
pairs who play against each other. The resulting
winners move on to the next round. The players
who lose are eliminated from the tournament.
From the rules above, it follows that if there are
N
players move on to
N players in a round, then
2
N+1
the next round if N is even and
players
2
move on to the next round if N is odd.
This process continues until the final round,
which is played between two players. The winner
in this round is the champion.
1.

Find the number of matches played by the champion.


A. In the first round, there were 169 players.
B. The champion was given a bye only once.

2.

The number of players in the first round was M


where 129 < M < 256. Find M.
A. One player received a bye while moving from
the third to the fourth round.
B. Only one player received a bye in the entire
tournament.

Directions for questions 3 to 6: Each question is


followed by two statements A and B. Indicate your
responses based on the following directives:
Mark (1)
Mark (2)
Mark (3)
Mark (4)
3.

if the question can be answered using A


alone but not using B alone.
if the question can be answered using B
alone but not using A alone.
if the question can be answered using A
and B together, but not using either A or
B alone.
if the question cannot be answered even
by using A and B together.

Class X has 80 students. The average height of


the students in it is 140 cm. It has two sections,
A and B, with equal number of students in each
section. The average height of A exceeds that of
B. Mohan is the tallest in A and Sohan is the

10

shortest in B. If each of these students is


transferred to the other section, the average
heights of the sections would get interchanged.
Find the height of Mohan.
(A) The average heights of A and B differ by 2 cm.
(B) If Sohan shifted from B to A, the average
heights of the sections would become equal.
4.

A company has to store at least 270 kilolitres of


water at all times to meet safety and regulatory
requirements. It is considering having a spherical
tank whose wall thickness is uniform and whose
outer radius is 6 meters for this purpose. Will the
tank meet the company requirements?
(A) When empty, the tank weighs 36000 kg. It is
made of a material whose density is 4 gm/cc.
(B) The tanks inner radius is at least 4.5 metres.

Directions for questions 3 to 8: Select the correct


alternative from the given choices.
3.

The price of coffee was increased by 40% but Raj


was willing to increase his expenditure by 12%.
Find by what percentage should he decrease his
consumption.
(A) 10%
(B) 15%
(C) 20%
(D) 25%

4.

The ratio of the populations of cities X, Y and Z in


2008 was 3 : 5 : 6. The percentage increases in
the populations of X, Y and Z from 2008 to 2009
were 10%, 12.5% and 15% respectively. Find the
percentage increase in their total population from
2008 to 2009.
(A) 12%
(B) 14%
(C) 13%
(D) None of these

5.

P, Q and R are three integers. Find the maximum


value of PQ + QR + PR.
(A) P = Q R
(B) P + Q + R = 84

5.

If the cost of a ball pen reduces by 20%, Raj can


buy 90 more ball pens for `3600. Find the cost (in
`) of a ball pen.
(A) 12.5
(B) 10
(C) 15
(D) 20

6.

P is a point on AB. Rohit wanted to draw a


square ABCD but failed to do so. Why did he fail?
(A) PC = 3 cm
PD
(B) PC =
3

6.

Ashok made a loss of 15% by selling 96 apples


for `2040. How many apples must he sell for
`2600 to make a 30% gain?
(A) 80
(B) 100
(C) 65
(D) 104

7.

Three persons A, B and C have their monthly


incomes in the ratio 6 : 7 : 8. Their monthly
expenditures are in the ratio 5 : 6 : 10. The monthly
savings of C is 37.5% of his monthly income. Find
what percent of Bs savings was As savings?
(A) 50%
(B) 662/3% (C) 831/5% (D) 87.5%

Percentage, Profit & Loss


Directions for questions 1 and 2: Answer these
questions based on the information below.
Mohan was considering three alternatives for investing
a certain amount. He wanted to get the maximum
possible assured return on his investment. The three
alternative are given below. He could make use of
each completely or partially along with the others.
Alternative 1: Invest in the mutual funds of PQR Ltd.
A rise in the stock market will result in a return of 8%
and a fall will result in a return of 10%.
Alternative 2: Invest in the mutual funds of RQP Ltd.
A rise in the stock market will result in a return of 5%
and a fall will result in return of 4%.
Alternative 3: Invest in a public sector bank which
promises a 0.4% return.
1.

Find the greatest assured return for Mohan.


2
(1) 0.4%
(2) 0.5%
(3)
%
3
5
%
(4) 0.8%
(5)
6

2.

Find the strategy which will maximize


guaranteed return to Mohan.
(1) 100% in alternative 3.
(2) Equal investment in each alternative.
(3) Investments in alternatives 1 and 2 in
ratio 1 : 2
(4) Investments in alternatives 1 and 2 in
ratio 2 : 1
(5) Investments in alternatives 1, 2 and 3 in
ratio 2 : 3 : 4

the

the
the
the

8. Pradeep bought a puppy for a certain price.


He sold it to his neighbour at a profit percent
whose magnitude was equal to the profit realised
by him in the transaction. But after a couple of
days the neighbour sold it back to Pradeep at
20% loss. Effectively, 27.5% of the cost price was
refunded to Pradeep. At what profit percentage
did he sell the puppy to his neighbour?
(A) 30%
(B) 37.5% (C) 27.5% (D) 32.50%

Simple Interest Compound Interest


Directions for questions 1 and 2: Select the correct
alternative from the given choices.
1.

The difference between the simple interest and


the compound interest for two years on a sum
invested at 16% p.a. is `384. Find the sum
(in `)
(A) 13500 (B) 15000 (C) 14250 (D) 12750

2.

Ramu took a certain loan at Simple Interest in


2000 for a period of 4 years. The rate of interest
was constant throughout the loan period
whereas had he cleared the loan after 9 years,
he would have paid `90,000. Where as had he
cleared the loan after 12 years, he would have
paid `105,000. Find the amount (in `) he paid to
clear the loan.
(A) 45000 (B) 65000 (C) 70000 (D) 75000

11

Time and Work

them working each day, the total time taken


would be 241/2 days. Find the number of days A
alone would take to complete the work if B is
faster than A.
(A) 28
(B) 21
(C) 24
(D) 25

Directions for questions 1 to 7: Select the correct


alternative from the given choices.
1.

Pipe X can fill a tank in a certain time. It was


opened at 12 pm. Due to a leak at the bottom of
the tank, the tank was filled only at 12:50 pm.
If the leak can empty the tank in 200 minutes,
then find the time (in minutes) in which X can fill
the tank.
(A) 30
(B) 45
(C) 40
(D) 50

2.

8 men can build a wall, 8 m long in 8 days,


working 8 hours a day. Find the number of
hours per day for which 16 men are required to
work, to build a wall 16 m long in 16 days.
(A) 2
(B) 8
(C) 16
(D) 4

3.

50 men can complete a job in 20 days working


9 hours a day. They started the job. They
worked 9 hours each day for the first x days.
At the end of x days, 5 men left. The remaining
job was completed by the remaining men
working 8 hours a day in 12.5 days. Find x.
(A) 12.5 (B) 8
(C) 9
(D) 10

4. If A, B, C, D work independently, the amount that


A is paid for 3 days is equal to that paid to B for
4 days. The amount paid to C for 3 days is equal
to that paid to D for 2 days. B and D are paid
equal amounts for equal duration. A and B
together complete a piece of work for which they
are paid `1680. If all 4 had completed the same
work together, what would A's share have been
(in rupees)?
Assume that equal amounts are paid for equal
work.
(A) 960
(B) 800
(C) 560
(D) 630
5. For workers working on a construction site, the
rate of doing work for men and women increases
in winter with respect to that in summer by
1
33 % and 50% respectively. 4 men and 12
3
women can complete a certain piece of work in
120 days in summer. The time taken triples if the
women do not turn up for the work. If a men and
b women can complete two times the given work
in 180 days in winter then find the possible
number of ordered pairs of (a, b).
(A) 2
(B) 20
(C) 10
(D) 5
6. A and B are two daily labourers who work on a
maintenance site. The daily wage of A is 40%
less than the daily wage of B. A and B together
worked on a certain project and completed it in
72 days. As a result, at the end of the project they
together received a certain amount. If that entire
amount were to be earned by A alone, then for
how many days would he need to work ?
(A) 144
(B) 216
(C) 288
(D) 192
7. A and B can together do a piece of work in
12 days. If A completed half the work and the
other half is completed by B with only one of

Venn diagrams
Directions for questions 1 to 3: Select the correct
alternative from the given choices.
1.

In a class, 30% of the students like tea and 40%


of the students like coffee. 20% of the students
who like tea also like coffee. Find the
percentage of the studies who like neither tea
nor coffee.
(A) 32% (B) 34%
(C) 38%
(D) 36%

2.

In a locality, 180 residents watch only Sony TV,


210 residents watch only Star Plus and 150
residents watch only Zee TV. 540 residents
watch atleast one of Sony TV and Star Plus. At
most 340 residents watch Star Plus. 90
residents watch all the three channels. Find the
minimum possible number of students who
watch Sony TV and Zee TV but not Star Plus.
(A) 40
(B) 50
(C) 20
(D) 30

3.

In a group, 60% of the boys and 50% of the girls


like cricket. 45% of the boys and 55% of the
girls like volleyball. Number of students who like
cricket is 7 more than that of students who like
volleyball. The difference of the number of boys
and girls who like cricket and that of the number
of boys and girls who like volleyball are in the
ratio 16 : 5. Find the strength of the group.
(A) 90
(B) 100
(C) 110
(D) 120

Trigonometry
Directions for questions 1 to 3: Select the correct
alternative from the given choices.
1. T1 and T2 are two towers and Raju was on the top
of the tower T1. He realized that there were two
points on the ground such that the angle of
elevation of T1s top from each of those points
was . The distances from, T2s bottom to the top
of T1 as well as to each of the points was
30 feet. The area of the triangle formed by the top
of T1 and each of the points can be (in feet).
(T2 's bottom and the two points on the ground
are collinear)
(A) 225
(B) 960
(C) Both (A) and (B)
(D) Neither (A) nor (B)
2. A ladder has a length of 10 m. It makes an angle
of 45 with a wall. It touches the wall at a point P.
There are two points on the ground. The angle of
elevation of P from each of these points is 60.
Find the distance between these points (in m).
(The two points on the ground and the bottom of
the wall are collinear)

(A)

7 6
3

(B)

10 6
3

(C)

8 6
3

(D)

4 6
3

12

pq + qr + rp
, where p, q, r are
p 2 + q2 + r 2
real numbers such that the sum of any two
exceeds the third, which of the following is not a
possible value of ?
(A) 20 (B) 40
(C) 50
(D) 80

3. Given that cos =

Indices, Logarithms, Surds


Directions for questions 1 to 7: Select the correct
alternative from the given choices.
1.

33x + 2 92x 1 = 118098, find the value of 6x + x6.


(A) 945
(B) 559
(C) 2403
(D) None of these

2.

Find the value of


1
17 16 + 16 17

(A)

3.

20
15

(B)

1
18 17 + 17 18

7
44

(C)

+ ... +

1
121 120 + 120 121

37
44

(D)

13
15

1
1
6
= 3, find the value of x + 6 .
x
x
(A) 729
(B) 326
(C) 322
(D) 324

If x +

4. If a = 5 2 6 and b = 5 + 2 6 , evaluate
4

a +b

a3 + b3

(A)
5.

4. If x1, x2, x3, x4, x5 and x6 are positive and x1 x2 x3


x4 x5 x6 = 1; then the minimum value of (x1 + 4)
(x2 + 4) (x3 +4) (x4 +4) (x5 +4) (x6 +4) is
(A) 15625
(B) 1425
(C) 13225
(D) 11025
1
1 1 1 1
, then S lies in
+ + + + ... +
512
1 2 3 4
which of the following ranges?
(A) (4, 5.5)
(B) (5.5, 9)
(C) (9, 10.5)
(D) (10.5, 12)

5. If S =

6. P is any positive number such that it is possible


to find 4 positive numbers whose product is P
and whose sum S P. Among the numbers 4, 5,
6, 7, how many are possible values of P?
(A) 0
(B) 1
(C) 2
(D) 3
7. If x3 + y3 + z3 = 125, a2 + b2 + c2 = 16, ax + by + cz =
20 and x, y, z, a, b, c are integers, then which of
the following cannot be the value of
a+b+c
?
4 x 2 + y 2 + z2

1
(A)
25

4901
49

(B)

4799
485

(C) 10

(D)

4801
485

If log2log4x = log4log2x, find x.


(A) 16

3. What is the set of values of x for which the


4x 2 9x + 8
> 3 is satisfied
inequation 2
x 5x 6
(A) (6, 1) (1,)
(B) (, 6) (1, )
(C) (, )
(D) (, 1) (6, )

(B) 8

(C) 8 2

(D) 4 2

6. If 1, log7(4x + 5), log7(4x + 1 1) are in arithmetic


progression, which of the following is a possible
value of x?
(A) 1
(B) 1.5
(C) 2
(D) 2.5
2
7. Let t = 3(log3a) 9log3a + 9. Which of the
following is true about the equation at=27?
(A) It has exactly one distinct solution for t.
(B) It has exactly two distinct solutions for t.
(C) It has exactly three distinct solutions for t.
(D) It has no real solutions for t.

Modulus / Inequalities
Directions for questions 1 to 10: Select the correct
alternative from the given choices.
1. How many integer values of x exist that do not satisfy
the inequality (x2 + 4x 32) (x2 + 2x 8) > 0?
(A) 7
(B) 8
(C) 6
(D) 4
2. Under which of the following conditions the
a2 b2

a b is true?
inequation
b
a
(A) a b and ab < 0
(B) a b and ab > 0
(C) Either (a) or (b)
(D) None of these

(B)

1
27

(C)

1
33

(D)

1
49

8. How many integral values of x satisfy the


following inequality
14 < x 5 + x + 3 + x + 7 < 25?

(A) 9

(B) 11

(C) 13

(D) 16

9. Find the area described by the inequality


x + y + xy 2

(A) 3

(B) 2

(C) 2.5

(D) 4

10. In triangle PQR, p, q and r are the lengths of the


sides opposite P, Q, and R respectively. If (p + q
+ r)2 = 3(pq + qr + rp), what can be said about the
triangle PQR?
(A) It is equilateral.
(B) It is isosceles.
(C) It is scalene.
(D) Cannot be uniquely determined

Operator Based Questions


Directions for questions 1 to 3: Select the correct
alternative from the given choices.
1. Given that a b = 4a + 3b + 7ab and a a < b
b, then which of the following is true?
(A) (a b) (a + b + 1) < 0
(B) (a + b) (a b + 1) > 0
(C) (a b) (a + b+ 1) > 0
(D) (a+ b) (a b + 1) > 0

13

2. If x @ y = y 4 x + + x
x

1
of 6 @
is
6
3891
(A)
36
4179
(C)
36

3.

(B) 1
(D)

1297
36

The operations * and are defined as


a * b = a + b + ab and a b = ab (a + b).
Find the value of (3 * 4) (4 5).
(A) 19 (29) (113)
(B) (85) (30)
(C) 19 (180) (199)
(D) 84 (29) (113)

Numbers
Directions for questions 1 to 43: Select the correct
alternative from the given choices.
1. Raju wrote the first 50 natural numbers one after
another on a black board. He then carried out the
following procedure 49 times. In each instance,
he erased two numbers, say p and q and
replaced them by a single number p + q 1.
Find the final number left on the board.
(1) 1224
(2) 1275
(3) 1276
(4) 1274
(5) 1226
2.

Find the last two digits of 73024.


(1) 41
(2) 81
(4) 01
(5) 61

(3) 21

3. N is a natural number. Div(N), a function of N, is


defined as follows:
Div(N) = N if N 9
= Div(S(N)), otherwise,
where S(N) is the sum of the digits of N.
For instance, Div(8) = 8,
Div(625) = Div(6 + 2 + 5) = Div(13) = Div(1 + 3)
= Div (4) = 4 etc.

Find the number of positive integer values of N


less than 600, for which Div (N) = 9.
(1) 44
(2) 55
(3) 66
(4) 77
(5) 67
4.

Find the value of

upto 2005 terms


1
(1) 2005
2006
1
(3) 2006
2005
1
(5) 2006
2007

1+

of
the
PQ=
(1) 2
(4) N

3
4 y + , then the value
y

resulting

even

elements,

then

(3) 1

(2) 1
(5) N1

6. X and Y are natural numbers. X is odd and less


than 100. Find the number of solutions of
1
1 3
=
.
X 18 Y
(1) 4 (2) 3 (3) 2 (4) 1 (5) 5
7. A tournament had 2N + 1 teams t1, t2, .. t2N+1
where N > 6. Each team had x players where
x > 4. The following pairs of teams have a
common player : t1 and t2N+1, t2 and t2N, .. tN and
tN+1. These are the only pairs of teams who have
a common player. Find the total number of
players in the 2N + 1 teams.
(1) N(x 1) + x
(2) N(2x 1) + x
(3) N(2x 2) + x
(4) N(x + 1) + x
(5) N(x + 2) + x
8. A four digit number has the form AABB. It is also
a perfect square. How many (A, B) are possible?
(1) 0 (2) 3 (3) 4 (4) 1 (5) 2
9.

Let M = 3(3!) + 4(4!) + .. + 15(15!)


What is the remainder when M 15 is divided by
14! 2?
(A) 14! 443
(B) 14! 459
(C) 459
(D) 443

10. A number when divided by 5, 6, 7 and 8 leaves


remainders of 3, 4, 5 and 6 respectively.
How many such 4-digit numbers are there?
(A) 11
(B) 10
(C) 9
(D) 12
11. If x =
1
32

(A)

1
1
1
+ 2 + 2 + .... , what is the value of
22
3
4
1
52

1
72

+ .... in terms of x?

3x
3x 1
x 1
1 (C)
(B)
4
4
2

12. If k =

49 80 42 80
49 79 + 42 79

(D)

x
1
2

, then

(A) 0 < k 1
(C) 4 < k 7

(B) 1 < k 4
(D) k > 7

1
1
1
1
+
+ 1+ 2 + 2 +
12 22
2
3

13. If A = 71421 . 98 105 112 ...189 196,


what is the remainder when A is divided by 9?
(A) 1
(B) 3
(C) 7
(D) 5

1
2006
1
(4) 2005
2005

14. If 2a + 5b = 7(a2b5)1/7 and x =

(2) 2006

5. Set A = {3, 4, .. 2N + 1, 2N + 2}, where N is a


natural number. Each odd element in it was
increased by 3 and each even element in it was
increased by 1. P denotes the average of the
resulting odd elements and Q denotes the average

a + b
, what is
2

the value of 2(x + a)2 + 5(x b)5?


(A) 7
(B) 168
(C) 3
(D) None of these

15. Which of the following is a rational number?

(A) (7 + 4 3 )50 + (7 4 3 )50

(B) (7 4 3 )50 + (7 + 4 3 )50


log

(C) 4 3 4 3 + 7 log7
(D) None of these

4 3

14

16. Which of the following is prime?


(A) 270 + 1
(B) 296 + 1
160
(C) 2 + 1
(D) None of these
17. A 200 page book is formed by using 50 sheets,
folded in the middle and stapled along the fold,
with each sheet providing 4 pages. The pages
are then numbered from 1 to 200. The nth sheet is
removed from the book. What is the sum of the
pages on this sheet?
(A) 400 2n
(B) 400 n
(C) 200 + 4n
(D) None of these
18. If u + v + w + x + y = 15, what is the maximum
value of uvx + uvy + uwx + uwy?
(A) 125 (B) 144
(C) 3125 (D) 243
10

19. The remainder when 1301 1301 is divided by


21 is
(A) 1
(B) 2
(C) 19
(D) 20
20. If a, b c, d are natural numbers such that
23 < a < b < 40 < c < d < 50, how many values
are possible for the quadruplet (a, b, c, d)?
(A) 4284 (B) 4320 (C) 4200 (D) 4165
21. A number N leaves a remainder of 7, 10 and
17 respectively when divided by 15, 21, and 35.
What is the remainder when N is divided by 105?
(A) 73
(B) 59
(C) 52
(D) 55
22. How many five digit numbers with digits that are
not necessarily distinct are divisible by 6 but not
by 12?
(A) 7500 (B) 7499 (C) 14999 (D) 15000
23. Which of the following numbers does not divide
(412 1)?
(A) 5
(B) 63
(C) 255
(D) 127
24. Find the number of factors of 243243 which are
multiples of 21.
(A) 20
(B) 21
(C) 22
(D) 24
25. What is the remainder when 4911 + 5011 + 5111 +
5211 is divided by 202?
(A) 0
(B) 101
(C) 201
(D) 1
26. Q is the number formed by knocking off all the
terminal zeros from 256!. What is the index of the
highest power of 12 that divides Q?
(A) 126
(B) 97
(C) 96
(D) None of these
27. Let N = (31)(32)(33)(98)(99)(100)
If N is divisible by 12n where n is a natural
number, find the maximum value that 'n' can take.
(A) 30
(B) 34
(C) 35
(D) 40
28. 'a' and 'b' are prime numbers and n is an integer
such that 1 n ab. What is the sum of all the
possible values of n such that n and ab are
coprime to each other?
ab a b + 1
(A) (a 1) (b 1)
(B)
2
ab(a 1)(b 1)
(C) ab
(D)
2

29. Set A = {1, 4, 7, 10, 13, , 100}. B is a non


empty proper subset of A such that all the
elements of B are divisible by 7. Find the number
of such subsets.
(A) 5
(B) 31
(C) 32
(D) 63
30. There are 4 distinct integers p, q, r and s, such
that the equation (n - p) (n - q) (n r) (n s) = 9
is satisfied for some integral values of n. How
many integral values of n satisfy the given
equation for a particular set of values of p, q, r,
and s?
(A) 0
(B) 1
(C) 2
(D) More than 2
31. Find the remainder when
4444433333) is divided by 7
(A) 0
(B) 2
(C) 3

(3333344444

(D) 6

32. Find the remainder when 4 + 44 + 444+ 4444+ ...


+ 444 . . . (50 digits) is divided by 9.
(A) 0
(B) 3
(C) 6
(D) 8
33. Consider the equation

1 1 1
+ = , where a and b
a b 6

are integers. How many ordered pairs (a, b) exist


which satisfy the given relation?
(A) 9
(B) 10
(C) 17
(D) 18
34. The difference between the squares of two
integers is 420. How many such pairs of integers
exist?
(A) 4
(B) 8
(C) 12
(D) 16
35. How many 9 digit multiples of 6 can be formed
using only the digits 8 or 9?
(A) 56
(B) 84
(C) 86
(D) None of these.
36. (a + b + c + d)5 (a5 + b5 + c5 + d5) is always
divisible by
(A) 24
(B) a + b + c + d
(C) 9
(D) 5
37. If A = 1! + 2! + 3! + 4! + .......+ 49! and
B = 1! + 2! + 3! + 4! +......+168!, then which of the
following is true?
(A) A is a perfect square.
(B) (B) is a perfect square.
(C) Both A and B are perfect square.
(D) Neither A nor B is a perfect square.
38. What is the sum of the even factors of 3600?
(A) 4030
(B) 12896
(C) 6046
(D) 12090
39. Consider all 5-digit numbers for which the sum of
the digits is 41. How many of these are divisible by
11?
(A) 10
(B) 12
(C) 16
(D) More than 16
40. In how many ways is it possible to express 36 as
a product of 3 positive integers?
(A) 8
(B) 6
(C) 12
(D) None of these

15

41. W is a whole number. The function root(W) is


defined as follows
root (W) = W if W 2
= root (W 3) if 3 W 9
= root (sum (W)) otherwise, where sum (W)
denotes the sum of the digits of W
For example
Root (7) = root (7 3) = root (4) = root (4 3)
= root (1) = 1
Root (245) = root (2 + 4 + 5) = root (11)
= root (1 + 1)= root (2) = 2
How many values of W less than 500 satisfy the
condition root (w) = 0?
(A) 165
(B) 166
(C) 167
(D) 168
42. The 18-digit number X has a 7 in the units place.
If this 7 is transposed to the front of the number,
the resulting 18-digit number is 5 X. Find X.
(A) 124587124587124587
(B) 124857124857124857
(C) 142857142857142857
(D) 142587142587142587
43. How many numbers less than or equal to 1108
are coprime to 252?
(A) 311
(B) 312
(C) 316
(D) 318

Directions for questions 44 to 46: These questions


are based on the following data.
Professor Calculus devised a game to pass his time.
The game was played in the following manner. He
kept 500 coins numbered from 1 to 500, on a table in
sequence all showing heads.
In round 1, he flipped all the coins such that each
showed a tail. In round 2, he flipped only those coins
whose position was a number divisible by 2.
In round 3, he flipped only those coins whose position
was a number divisible by 3 and so on.
He continued flipping the coins for 500 rounds.
44. What was the greatest number of consecutive
coins all showing heads at the end of the game?
(A) 16
(B) 41
(C) 42
(D) 43
45. Among the following coins, which coin was
flipped the greatest number of times?
(A) 210
(B) 324
(C) 288
(D) 240
46. How many coins did Professor Calculus flip in
round 13, which were flipped only once in the
previous rounds?
(A) 7
(B) 8
(C) 11
(D) 38

Directions for questions 47 to 49: Select the


correct alternative from the given choices.
n
47. If the sum of all integers between 3 and
n+3
3 (both exclusive) is divisible by 70 where n is a
positive integer, then which of the following is
necessarily true about n?
(A) It is divisible by 4
(B) It is a multiple of 6
(C) It is divisible by 7
(D) It is divisible by 5.

48. Find the units digit in 13 + 23 + 33+ +903.


(A) 0
(B) 5
(C) 6
(D) 1
49. If S(N) denotes the sum of the digits of N, then
what is the remainder when S(1) + S(2) + S(3) +
..+ S(99) is divided by 99?
(A) 0
(B) 9
(C) 18
(D) 36

Probability
Directions for questions 1 to 4: Select the correct
alternative from the given choices.
1. There are 50 tokens numbered 1 to 50 kept on a
table. You are asked to pick two tokens at
random from the table one after the other. What
is the probability that the difference between the
numbers on the tokens picked by you lies
between 1 and 6 (both excluded)?
36
186
(A)
(B)
245
1225
372
186
(D)
(C)
2450
1225

2. A number is selected at random from 1 to 105.


Consider the following 3 events.
P: X is coprime to 3`
Q: X is coprime to 5
R: X is coprime to 7
Which of the following is true about the events
P, Q, R?
(A) They are pairwise independent and mutually
exclusive.
(B) The are pairwise independent but not
mutually exclusive.
(C) They are not pairwise independent but are
mutually exclusive.
(D) They are neither pairwise independent nor
mutually exclusive.
3. Alsi Arora worked in a private company where the
working hours were from 9:00 am to 6 pm. In a
year on 25% of the days he arrived late to office
whereas on 35% of the days he left early from
office. If p is the probability of the number of days
that he worked for the entire working day, then
which of the following best describes the possible
values of p?
(A) P = 0.4875
(B) 0.25 P 0.65
(C) 0.25 P 0.65
.
(D) 0.40 P 0.65
4. Ramesh doesn't always have time to read the
daily newspaper and whenever he reads it he
does it either in the morning or in the evening.
The probability that he reads it in the morning is
10% and the probability that he reads it in the
evening is 30%. Find the expected number of
days in a month of 30 days on which he doesn't
read the paper at all.
(A) 16
(B) 18.9
(C) 21
(D) 20.5

16

Miscellaneous

Inequations

Directions for question 1: Select the correct


alternative from the given choices.

Directions for question 1: Select the correct


alternative from the given choices.

1. There is a sequence of 7 sets A1, A2, A4, each


consisting of 6 elements and another sequence of
n sets B1, B2, ,,,, Bn each consisting of 3 elements.
The union of all the A's is equal to the union of all
the B's. If each element of this union occurs in
exactly 3 of the A sets and 6 of the B sets, then
find n.
(A) 28
(B) 21
(C) 14
(D) Cannot be determined

Coordinate Geometry
Directions for question 1: Select the correct
alternative from the given choices.
1.

If the equation 9x2 y2 + 6y = A where A is a


constant, represents a pair of straight lines, then
which of the following gives the point of
intersection of those two lines?
(A) (1, 2)
(B) (0, 3)
(C) (0, 3)
(D) cannot be determined

1. A triangle with sides x, y and z is such that


x3 + y3 +z3 = 3xyz. Which of the following is true
regarding the triangle?
(A) It is an isosceles triangle
(B) It is an equilateral triangle
(C) It is an obtuse angled triangle
(D) It is a right angle triangle

Statistics
Directions for question 1: Select the correct
alternative from the given choices.
1. The median of 11 numbers is x. If the greatest
number is removed the median of the remaining
numbers is 10.5. If the smallest is removed the
median of the remaining numbers is 13. If x (or
one of the numbers which is equal to x) is
removed the median would be
(A) 11
(B) 11.5
(C) 12
(D) Cannot be determined

17

Vous aimerez peut-être aussi